2019 AMC 10A Problems/Problem 19

Revision as of 18:05, 9 February 2019 by P groudon (talk | contribs) (Created page with "==Problem== What is the least possible value of <cmath>(x+1)(x+2)(x+3)(x+4)+2019</cmath>where <math>x</math> is a real number? <math>\textbf{(A) } 2017 \qquad\textbf{(B) } 2...")
(diff) ← Older revision | Latest revision (diff) | Newer revision → (diff)

Problem

What is the least possible value of \[(x+1)(x+2)(x+3)(x+4)+2019\]where $x$ is a real number?

$\textbf{(A) } 2017 \qquad\textbf{(B) } 2018 \qquad\textbf{(C) } 2019 \qquad\textbf{(D) } 2020 \qquad\textbf{(E) } 2021$

Solution

See Also

2019 AMC 10A (ProblemsAnswer KeyResources)
Preceded by
Problem 18
Followed by
Problem 20
1 2 3 4 5 6 7 8 9 10 11 12 13 14 15 16 17 18 19 20 21 22 23 24 25
All AMC 10 Problems and Solutions

The problems on this page are copyrighted by the Mathematical Association of America's American Mathematics Competitions. AMC logo.png